LSAT and Law School Admissions Forum

Get expert LSAT preparation and law school admissions advice from PowerScore Test Preparation.

User avatar
 Dave Killoran
PowerScore Staff
  • PowerScore Staff
  • Posts: 5853
  • Joined: Mar 25, 2011
|
#83853
Side note: as with all questions in any of the Bibles, the more comprehensive discussion of this question is in the Forum for the test this question was drawn from, in this case at: viewtopic.php?f=690&t=14021&p=34551.

that said, let's look at your specific questions here:
leslie7 wrote:I'm just wondering here, (and it may have been touched on in the book already) but when we write
C (Depression)--> E Glasses since there is an arrow does this represent both a causal and conditional? (as in they can co-exist in a representation?)
No, I make mention of it in the book (page 296) that we use the same arrow in both instances but that they are very different (page 290). whenever you see it used as causal in the book you will see "C" and "E" above each term : )



leslie7 wrote:Also, are there ever instances when a causal relationship is not conditional? do we have examples of what that looks like and how we would write/understand it?
This is to some extent a thread throughout this entire chapter, and is also explicitly delineated on page 290. However, to really cement that idea, go look at the indicators you see for conditionality and then compare them to causality indicators. Very different words, with causal indicators being far more "Active" in nature (see point 3 on page 291). And, there are more examples of both ideas coming within the book!

These are both ideas that take a bit of getting used to in a formalized environment, so for now examine each problem we present and why we determine it's causal or conditional (or neither). you can have both ideas in the same stimulus, but that doesn't happen as much as each individually, and again the language would tell you when it was occurring.

Thanks!
 leslie7
  • Posts: 73
  • Joined: Oct 06, 2020
|
#83864
ok thanks a lot! sorry for not checking the right thread, there wasn't a reference to which LSAT test these questions came from in the book so I wasn't sure which LSAT test to look at. Do you have suggestions for figuring that out if it's not in the text? I appreciate your time!
User avatar
 Dave Killoran
PowerScore Staff
  • PowerScore Staff
  • Posts: 5853
  • Joined: Mar 25, 2011
|
#83886
leslie7 wrote: Sat Feb 06, 2021 6:31 am ok thanks a lot! sorry for not checking the right thread, there wasn't a reference to which LSAT test these questions came from in the book so I wasn't sure which LSAT test to look at. Do you have suggestions for figuring that out if it's not in the text? I appreciate your time!
There actually is--it's in the appendix at the end of the book! And it's online also on the book site :) We wouldn't leave out that kind of key info!

Thanks!
 ltowns1
  • Posts: 61
  • Joined: May 16, 2017
|
#94271
Was looking in the LR BIBLE and I came across this question. I think it’s really interesting. I picked (B) over ((E) but I see why it’s wrong. My main problem with (E) is that it feels that it still implies some small bit of causality by saying that glasses can impact depression, if only by a small amount. And if that were the case, wouldn’t that imply a reversal. Im clear on the fact that it’s the best answer, but not so clear on that point. Am I reading that wrong?
 ltowns1
  • Posts: 61
  • Joined: May 16, 2017
|
#94272
Just saw this wasn’t the right thread, I’ll put it in the correct one.
 Adam Tyson
PowerScore Staff
  • PowerScore Staff
  • Posts: 5153
  • Joined: Apr 14, 2011
|
#94275
The conclusion is that people wear the glasses because they are depressed or hypochondriacal. The condition is the cause and wearing glasses is the effect.

Answer E is saying the author assumes that it is not the other way around, that the glasses are not causing the depression. That's a classic Causal Assumption answer, ltowns1! Try the negation test on it; if wearing the glasses causes depression, that means the author got the relationship wrong, ruining their argument.
 ltowns1
  • Posts: 61
  • Joined: May 16, 2017
|
#94281
Hi, thanks for answering my questions. I should’ve been more clear, I’m aware of what (E) is doin conceptually. Original argument cause : D or H can cause people to wear tinted glasses. So I get that (E) from the authors perspective would make that connection stronger by saying that glasses don’t cause a persons mood to be depressed substantially. I’ve done the negation technique, but I guess the thing I’m hung up on is the word “substantially”, does that not imply that there still is a small connection in terms of glasses impacting mood?
 ltowns1
  • Posts: 61
  • Joined: May 16, 2017
|
#94368
To me this answer is talking about the dimness of the tint in the glasses, and how it doesn’t depress someone that much. That doesn’t equate to “tinted glasses do not cause depression” to me. I don’t know why. Please help my poor confused brain. Thank you!!

Katya
[/quote]

Hi so I was looking at this thread in relation to the question i had, and I noticed that Katya had the exact same thinking that I did about (E) in case my question came off as confusing. (E) seems like it's saying tinted glasses don't depress someone too much, and that was enough to give me doubt about the question. At the same time I should've recognized that (B) was wrong because answer choices that talk about what a person 'thinks' are hardly ever correct from my experience.
 Adam Tyson
PowerScore Staff
  • PowerScore Staff
  • Posts: 5153
  • Joined: Apr 14, 2011
|
#94371
Answer E is best understood by studying the negation, ltowns1. What if wearing the glasses DOES "depress the wearer's mood substantially"? If that was true, then the correlation between wearing the glasses and being depressed would suggest a different causal relationship than the one the author asserted. The glasses would appear to be the cause of, rather than an effect of, their depression, and that would ruin the author's argument. Since negating this ruins the argument, we know the author must believe that this is true.

The "substantially" doesn't hurt the answer, because the author must believe that any such effect is not substantial. Might they also have to believe that any such effect is completely non-existent? Maybe, but that doesn't make this assumption any less true. The correct answer doesn't have to prove that the author is correct; that's the job of a Justify the Conclusion answer, not an Assumption answer. Rather, the argument has to prove that the author must believe the answer choice. An Assumption answer is something that must, in the author's mind be true, and answer E fits the bill.
 ltowns1
  • Posts: 61
  • Joined: May 16, 2017
|
#94376
Thank you Adam.

Get the most out of your LSAT Prep Plus subscription.

Analyze and track your performance with our Testing and Analytics Package.